Chapter 17 Quiz 1-3

¡Supera tus tareas y exámenes ahora con Quizwiz!

(5) in order to ensure proper electrolyte distribution and maintain the polarity of the cell membrane, the sodium - potassium pump: A) moves two potassium ions into the cell for every tree sodium ions it moves out. B) moves two sodium ions into the cell for every three potassium ions it moves out of the cell C) moves three sodium ions and three potassium ions back in to the cell D) moves calcium and potassium ions back in to the cell by a process called passive transport.

A Electrophysiology, page 969

On the 12-lead ECG, extreme right axis deviation is characterized by: A)a negative QRS in lead I and a negative QRS in lead aVF B) a negative QRS in lead I and a positive QRS in lead aVF C) a positive QRS in lead I and a negative QRS in lead aVF D) a positive QRS in lead I and a positive QRS in lead aVF

A Page 1027

A STEMI should be suspected in a 45-year-old female if the ST segemnts in leads V2 and V3 are elevated by ____ or more. A) 1.5 B) 0.5 C) 1.0 D) 0.25

A Page 1031

When monitoring a patient's cardiac rhythm, it is important to remember that: A) the ECG does not provide data regarding the patient's cardiac output B) the presence of a QRS complex correlates with the patient's pulse C) a heart rate below 60 beats per minute must be treated immediately D) most patients with acute myocardial infarction experienceq asystole

A Page 1045

Which of the following clinical findings is LEAST suggestive of left-sided heart failure? A) Sacral edema B) Crackles in the lungs C) Shortness of breath D) An S3 gallop

A Page 967

A wandering atrial pacemaker A) may have variable PR intervals B) is generally treated with atropine C) is generally faster than 100 beats/min D) has consistent P wave shapes

A page 1000-1001

A regular rhythm with inverted P waves before each QRS complex, a ventricular rate of 70 beats/min narrow QRS complexes, and a PR interval of 0.16 seconds should be interpreted as an: A) accelerated junctional rhythm B) junctional escape rhythm C) ectopic atrial rhythm D) suraventricular tachycardia

A page 1002-1003

Ventricular bigeminy occurs when: A) every second complex is a PVC B) at least two differently shaped PVCs occur C) a 6 second strip contains at least two PVCs D) two premature ventricular complexes (PVCs) occur in a row

A page 1005

What is the R-on-T phenomenon? A) A PVC that occurs when the ventricles are not fully redepolarized B) A PVC that occurs during a time when the ventricles are depolarizing C) When the R wave occurs at the J point of the next cardiac cycle D) A unifocal PVC that occurs during the upslope of any given T wave

A page 1005

A 39-year-old in asystole has been unresponsive to high-quality CPR and two doses of epinephrine. The patient is intubated and an IO catheter is in place. You should focus on: A) searching for reversible causes B) transcutaneous cardiac pacing C) establishing a peripheral IV line D) providing mild hyperventilation

A page 1014-1017

A first-degree heart block has a PR interval greater than 0.20 seconds because: A) each impulse that reaches the AV node is delayed slightly longer than expected B) depolarization of the atria occurs at a slightly slower rate than one would expect C) the primary pacemaker is not the SA node, but rather an ectopic atrial pacemaker D) impulses generated by the SA node traverse the AV node at an accelerated rate

A page 1020

(7) Left ventricular hypertrophy should be considered in a 50-year-old patient if the sum of the depth of the S wave in lead V1 and height of the R wave in either lead V5 or V6 exceeds: (quiz 2) A) 35 mm B) 32 mm C) 22 mm D) 28 mm

A page 1029

You are evaluating a 60 year old woman's 12 lead ECG and note that the R wave height in lead V1 exceeds the S wave depth. what conditions would MOST likely cause this? A) Pulmonary hypertension B) Left ventricular failure C) Tricuspid valve murmur D) severe systemic hypertension

A page 1029

You are evaluating a 60-year-old woman's 12 lead ECG and note that the R wave height in lead V1, exceeds the S wave depth. What condition would MOST likely cause this? A) pulmonary hypertension B) severe systemic hypertension C) a tricuspid valve murmur D) left ventricular failure

A page 1029

Cardiac arrhythmia following an acute myocardial infarction: A) tend to originate from ischemic areas around the infarction B) generally originate from the center of the infarcted tissues C) typically manifest as atrial fibrillation or atrial tachycardia D) are uncommon within the first 24 hours after the infarction

A page 1041

Stable angina: A) occurs after a predictable amount off exertion B) usually requires both rest and nitroglycerin to subside C) typically subsides within 10 to 15 minutes D) is characterized by sharp chest pain rather than pressure

A page 1041

A 56-year-old man presents with an acute onset chest pressure, shortness of breath, and diaphorsis. He has a history of hypertension and type 2 diabetes. His airway is patient and his breathing is adequate. You should: A) administer supplemental oxygen B) obtain baseline vital signs C) acquire a 12-lead ECG tracing D) establish vascular access

A page 1044-1045

In a patient with left ventricular failure and pulmonary edema: A) increases pressure in the left atrium and pulmonary veins forces serum out of the pulmonary capillaries and in to the alveoli B) an acute myocardial infarction or chronic hypertension causes the left ventricle to pump against decreased afterload, resulting in hyopprefusion C) the right atrium and ventricle pump against lower pressures, resulting in the systemic pooling of venous blood D) diffusely collapsed alveoli cause blood from the right side of the heart to bypass the alveoli and return to the left side of the heart

A page 1050-1051

Which of the following mechanisms causes hypertension? A) increased afterload stimulates the Frank-Starling reflex, which raises the pressure behind the blood leaving the heart B) Atreiosclerosis results increased elasticity of the arteries, causing vasodilation and increased arteriolar capacity C) atherosclerotic palque narrows one or more of the coronary arteries, resulting in increased cardiac perfusion D) heart rate that is persistently about 80 beats/min causes an increased in cardiac output and a resultant increase in blood

A page 1058

(3) if a patient's aortic aneurysm is not compressing on any adjacent structures: (quiz 2) A) he or she will likely be asymptomatic B) the pain is often confined to the back C) a pulsatile mass will usually be absent D) his or her pain will likely be loss severe

A page 1063

Thousands of fibrils that are distrusted throughout the inner surfaces of the ventricles, which represent the end of the cardiac conduction system, are called the: A) purkinje fibers B) internodal pathways C) cardiac myocytes D) bundle branches

A page 971

The duration of the QRS complex should be _____ milliseconds or less in healthy adult A) 110 B) 120 C) 100 D) 130

A page 983

When analyzing a cardiac rhythm strip in lead II, you should routinely evaluate all of the following components, EXCEPT the: A) ST segment B) PR interval C) R-R interval D) QRS width

A page 985

A regular cardiac rhythm with a rate of 104 beats/min, upright P waves, a PR interval of 0.14 seconds, and QRS complexes that measure 0.10 seconds should be interpreted as: A) sinus tachycardia B) normal sinus rhythm C) junctional tachycardia D) supraventricular tachycardia

A page 991-992

(10) patients with a heart rate greater than 150 beats/min usually become unstable because of: (quiz 2) A) reduced ventricular filling B) an increase in the atrial kick C) a significantly reduced afterload D) increased right atrial preload

A page 993

Which of the following statements regarding an idioventricular rhythm is correct? A) Most patients with an idioventricular rhythm are hemoynamically unstable B) Treatment for an idioventricular rhythm focuses on increasing blood pressure C) Idioventricluar rhythms are typically accompanied by nonconducted P waves D) the most common cause of an idioventricular rhythm is failure of the SA node

A pages 1005-1006

On the ECG strip, third-degree AV block usually appears as a: A) slow, wide QRS complex rhythm with inconsistent PR intervals B) narrow QRS complex rhythm with a rate less than 60 beats/min C) slow, narrow QRS complex rhythm with irregular P-P intervals D) wide QRS complex rhythm with a rate between 50 and 70 beats/min

A pages 1021-1022

Your partner is evaluating a patient's 12-lead ECG and states that she sees a right bundle branch block. what did you partner observe? A) QRS complexes of 128 milliseconds B) a QRS pattern in lead V1 C) QRS complexes of 112 miliseconds D) a QS pattern in lead V1

A pages 1027-1028

Abnormal neurological signs that accompany hypertensive encephalopathy occur when: A) pressure causes a breach in the blood-brain barrier and fluid leaks out, causing an intracranial pressure B) neurons sustain permanent damage secondary to single increase in blood pressure above 200/130 mm Hg C) pressure in the brain causes transient dysfunction of the parietal lobe and cerebral vasodilation D) the mean arterial pressure exceeds 100 mm Hg and blood is forced from the brain and into the spinal cord

A pages 1058-1059

The second half of the T wave: A) represents a state of absolute ventricular refractoriness in which another impulse cannot cause depolarization B) is the point of ventricular repolarization to which a defibrillator is synchronized to deliver electrical energy C) represents a vulnerable period during which a strong impulse could cause depolarization, resulting in a lethal arrhythmia D) is the strongest part of ventricular depolarization is often the origin of dangerous ventricular arrhythmia

A 985

Which of following occurs at the AV node during a third-degree heart block? A) all impulses are blocked from entering the ventricles B) every third impulse is allowed to enter the ventricles C) there is an abnormal delay in conducting impulses D) impulses bypass the AV node and enter the ventricles

A page 1021

(3) in context to cardiac compromise, syncope occurs due to: A) a sudden cardiac dyshythmia B) a drop in cerebral perfusion C) an increase in vagel tone D) an acute increase in heart rate

B Patient Assessment, page 965

Once a advanced airway device has been inserted into a cardiac arrest patient: A) the compressor should pause so ventilations can be given B) ventilations are delivered at a rate of 10 breaths/min C) chest compressions should be increased to 120 per minute D) you should deliver one breath every 5 to 6 seconds

B page 1016

(8) a first-degree heart block has a PR interval greater than 0.20 seconds because: (quiz 2) A) the primary pacemaker is not the SA node, but rather an ectopic atrial pacemaker B) each impulse that reaches the AV node is delayed slightly longer than expected C) impulses generated by the SA node traverse the AV node at an accelerated rate D) depolarization of the atria occurs at a slightly slower rate than one would expect

B page 1020

The QRS in lead I is a positive deflection and QRS in lead aVF is a negative deflection. This indicates: A) a normal axis B) left axis deviation C) right axis deviation D) extreme right axis deviation

B page 1027

(9) leads V1 to V3 allow you to view the ____ wall of the left ventricle (quiz 2) A) lateral B) anteroseptal C) septal D) anterior

B page 1032

A 60-year-old man with crushing chest pain has 3 mm of ST elevation in leads V1 through V4. what should suspect? A) Right coronary artery occlusion B) left anterior descending artery occlusion C) left circumflex occlusion D) right ventricular infarction

B page 1032

Acute coronary syndrome is a term used to describe: (quiz 2) A) a sudden cardiac rhythm disturbance that causes a decrease in cardiac output B) any group of clinical symptoms consistent with acute myocardial ischemia C) acute chest pressure or discomfort that subsides with rest or nitroglycerin D) a clinical condition in which patients experience chest pain during exertion

B page 1039

Infarctions of the inferior myocardial wall are MOST often caused by: A) a blocked left anterior descending artery B) occlusion of the right coronary artery C) blockage of the left coronary artery D) acute spasm of the circumflex artery

B page 1041

A 59-year-old female presents with severe substernal chest pain. She is anxious and diaphoretic. what should you do? A) administer nitroglycerin B) administer aspririn C) obtain a 12 lead ECG D) establish IV access

B page 1045

Percutaneous coronary interventions involve: A) passing a 2 mm catheter through the femoral artery and administering a fibrinolytic agent through the catheter B) re-canalizing a blocked coronary artery by passing a balloon or stent through a catheter via periphearal artery C) passing a guide wire through one of the external jugular veins to directly visualize an occluded coronary artery D) using a large vein from one of the lower extremities to reroute blood flow past occluded coronary artery

B page 1045

Percutaneous coronary interventions involve: (quiz 2) A) using a large vein from one of the lower extremities to reroute blood flow past an occluded coronary artery B) recanalizing a block coronary artery by passing a balloon or stent through a catheter via peripheral artery C) passing a 2 mm catheter through the femoral artery and administering a fibrinolytic agent through the catheter D) passing a guide wire through one of the external jugular veins to directly visualize an occluded coronary artery

B page 1045

Nitroglycerin is contraindicated for patients: A) who are also taking beta-blocker medication B) with suspected with right ventricular infarction C) with a systolic BP less than 110 mm Hg D) with suspected left circumflex artery occlusion

B page 1047

Which of the following interventions would MOST likely performed en route to the hospital during a lengthy transport of a patient with a suspected myocardial infarction? A) Aspirin administration B) IV therapy and analgesia C) 12-lead electrocardiography D) Supplemental oxygen

B page 1048

You are dispatched to a residence at 2:00 am for an elderly man with shortness of breath. the patient tells you that he was suddenly awakened with the feeling that he was smothering. you notice dried blood on his lips. The patient tells you that he has some type of "breathing problem" for which he uses a prescribed inhaler and takes a "heart pill". you should suspect: A) right ventricular failure B) left ventricular failure C) reactive airway disease D) acute COPD exacerbation

B page 1050-1053

A 68-year-old male who has COPD presents with edema to his feet and ankles, jugular venous distention, and an enlarged abdomen. What should you suspect? A) Acute renal failure B) Right ventricular failure C) Acute hepatic failure D) Left ventricular failure

B page 1051

In addition to supplemental oxygen, treatment of a patient with left ventricular failure includes: A) a saline lock and a selective beta-2 adrenergic medication B) an Iv of normal saline to keep the vein open and nitroglycerin C) an IV of normal saline, a 20-mL/kg fluid bolus, and a diuretic medication D) a saline lock, fentanyl, and intubation facilitated by pharmacologic agents

B page 1054

The treatment for cardiogenic shock is generally focused on: A)increasing the heart rate to improve cardiac output and cerebral perfusion B) strengthening cardiac contractility without increasing the heart rate C) administering nitroglycerin to improve perfusion to the myocardium D) infusing enough IV fluid to maintain a systolic BP of 110 mm Hg.

B page 1057

If the paramedic must initiate drug therapy in the field for hypertensive encephalopathy, he or she should: A) first ensure that the heart rate is than 100/min B) maintain the patient in a supine position C) administer a 500 mL normal saline bolus first D)measure the BP at least every 5 to 10 minutes

B page 1059

A 60 year-old-women presents with fever, chills, and shortness of breath. She has a history of mitral valve prolapse. Assessment reveals flat, painless red lesions on the palms of her hands. What should you suspect? A) Cardiac tamponade B) endocarditis C) Myocarditis D) Pericarditis

B page 1060

Disruption of blood flow through the innominate artery due to dissection is likely to produce: A) collapsed jugular veins B) pulse or blood pressure deficits C) widened pulse pressure D) a rapid, irregular pulse

B page 1063

The left main coronary artery subdivides into the: A) left anterior ascending and descending arteries B) left anterior descending and circumflex arteries C) right coronary and left posterior descending arteries D) left posterior ascending and circumflex arteries

B page 963

A pulse that alternates in strength from one beat to the next is called: A) pulsus paradoxus B) pulsus alternans C) paradoxical pulse D) pulse deficit

B page 967

(11) the _____ represents the end of ventricular depolarization and the beginning of repolarization (quiz 2) A) T wave B) J point C) ST segment D) T-P interval

B page 984

Electrical capture during tanscutaneous cardiac pacing is characterized by: A) narrow QRS complexes that are preceded by a pacemaker spike B) a pacemaker spike followed by a wide QRS complex C) low-amplitude QRS complexes preceded by a pacemaker spike D) the presence of a strong pulse, despite a slow rate

B page 990

When assessing an anxious patient who presents with tachycardia, you must: A) prepare for cardioversion if the rate is less than 150 beats/min B) determine if the tachycardia is causing hemodynamic instability C) administer diazepam or midazolam to facilitate your assessment D) obtain a 12 lead ECG tracing before initiating any treatment

B page 991-992

The area of conduction tissue in which electrical activity arises at any given time is called the: A) myocyte B) pacemaker C) bundle of His D) sinus node

B page, 971

A regular rhythm with inverted P waves before each QRS complex, a ventricular rate of 70 beats/min, a narrow QRS complexes, and PR interval of 0.16 seconds should be interpreted as an: A) accelerated junctional rhythm B) junctional escape rhythm C) supraventricular tachycardia D) ectopic atrial rhythm

B pages 1002-1003

A pathologic Q wave: A) is deeper than one-quarter of the height of the R wave and indicates injury B) is wider than 0.04 seconds and indicates that a myocardial infarction occurred in the past C) generally indicates that an acute myocardial infarction has occurred within the past hour D) can only be substantiated by viewing at least two previous 12-lead ECGs

B pages 1031

(4) In contrast to the right side of heart, the left side of the heart: (quiz 1) A. Is relatively low-pressure pump that must stretch its walls in order to force blood through the aorta B. Drives blood out of the heart against the relatively low resistance of the pulmonary circulation C. Drives blood out of the heart against the relatively high resistance of the systemic circulation. D. Is a high-pressure pump that sends blood through the pulmonary circulation and to the lungs

C Anatomy and Physiology review, page 963

An electrical impulse is slightly delayed at the AV node so that the: A) bundle of His an depolarize fully B) primary cardiac pacemaker can reset C) atria can empty into the ventricles D) ventricles can contract completely

C Electrophysiology, page 971

If a particular interval on the ECG graph paper is 1.5 small boxes in width, the interval would be measured as: A) 600 milliseconds B) 45 milliseconds C) 0.06 seconds D) 2 millimeters

C page 982

A wide QRS complex that is preceded by a normal P wave indicates: A) rapid conduction through the ventricles B) a delay in conduction at the AV junction C) an abnormality in ventricular conduction D) that the rhythm is ventricular in origin

C page 982-983

A prolonged PR interval: A) is a sign of rapid atrial depolarization B) is greater than 120 milliseconds C) indicates an abnormal delay at the AV node D) indicates that the AV node was bypassed

C page 983

Supraventricualr tachycarida is MOST accurately defined as: A) an irregular tachycardic rhythm that originates just below the AV junction B) any tachycardic rhythm with a heart rate greater than 130 beats/min and absent P waves C) a tachycardic rhythm originating from a pacemaker site above the level of the ventricles D) a regular tachycardic rhythm between 150 and 180 beats/min with P waves buried in the QRS complexes

C page 993

An unresponsive, pulse less, apneic patient presents with ventricular tachycardia on the cardiac monitor. After defibrillating the patient, you should: A) deliver another shock if his cardiac rhythm is unchanged B) take no more than 10 seconds to assess for a pulse C) resume CPR and reassess the patient after 2 minutes D) reassess the cardiac rhythm to see if it has changed.

C pages 1014-1016

Left bundle branch block is characterized by: A) a QRS of less than 120 milliseconds and a terminal R wave in lead V1 B) a QRS of less than 120 milliseconds and a terminal S wave in lead V1 C) a QRS of greater than 120 milliseconds and a terminal S wave in lead V1 D) a QRS of greater than 120 milliseconds and a terminal R wave in lead V1

C pages 1027-1028

(12) a concordant precordial pattern exists when all QRS complexes: (quiz 2) A) in V1 through V6 end with a R wave B) are less than 120 milliseconds C) are upright in leads V1 through V6 D) are greater than 120 milliseconds

C pages 1028-1029

(2) An electrical wave moving in the direction of a positive electrode will: (quiz 3) A) manifest with narrow QRS complexes B) cause a negative deflection on the ECG C) cause a positive deflection on the ECG D) produce a significant amount of artifact

C pages 982

An early complex that breaks the regularity of the underlying rhythm and that is characterized by a narrow QRS complex and an upright P wave that differs in shape and size from the P waves of the other complexes MOST accurately describes an: A) junctional escape complex B) wandering atrial pacemaker C) premature atrial pacemaker D) atrial escape complex

C pages 993

Q waves are considered abnormal or pathologic if they are: A) present in a patient who is experiencing chest pressure or discomfort B) not visible in leads I or II when the QRS gain sensitivity is increased C) more than one-third the overall height of the QRS complex in lead II D) greater than 0.02 seconds wide and consistently precede the R wave

C page 983

(1) cardiac-related chest pain may be palliated by: (quiz 1) A. Mild exercise B. Nitroglycerin. C. Stress D. Exertion

B. Nitroglycerin Patient Assessment, page 964

(2) The farther removed the conduction tissue is from the SA node: (quiz 1) A. The longer the PR interval will be B. The faster its intrinsic rate of firing C. The narrower the QRS complex will be D. The slower its intrinsic rate of firing.

D Eletrophysiology, page 972

Junctional escape rhythms are characterized by: A) QRS complexes greater than 0.12 seconds B) inverted P waves before the QRS complex C) an absence of P waves D) a ventricular rate of 40 to 60 beats/min

D page 1002

(5) the brief pause between the P wave and QRS complex represents: (quiz 2) A) depolarization of the inferior part of the atria B) full dispersal of electricity throughout both atria C) the period of time when the atria are repolarizing D) a momentary conduction delay at the AV node

D page 983

In males, the QT interval is considered prolong if is ______ milliseconds or longer. A) 430 B) 400 C) 390 D) 450

D page 985

Electrical capture during transcutaneous cardiac pacing is characterized by: (quiz 2) A) low-amplitude QRS complexes preceded by a pacemaker spike B) the presence of a strong pulse, despite a slow rate C) narrow QRS complexes that are preceded by a pacemaker spike D) a pacemaker spike followed by a wide QRS complex

D page 990

(1) In contrast to the pain associated with acute myocardial infarction, pain from a dissecting aortic aneurysm:(quiz 3 ) A) often waxes and wanes B) is preceded by other symptoms C) gradually becomes severe D) is maximal from the onset

D page, 1063

Which of the following statement regarding second degree heart block is correct? A) More than half of all second-degree heart blocks cause hemodynamic compromise and require transcutaneous cardiac pacing in the prehospital or hospital setting B) Second-degree heart block is characterized by inconsistent PR intervals, a QRS complex greater than 0.12 seconds, and a ventricular rate less than 40 beats/min C) Most second degree heart block are transient in nature and resolve in the preshospital setting without the need for intervention in the emergency department D) Second-degree heart block occurs when an impulse reaching the AV node is occasionally prevented from proceeding to the ventricles and causing a QRS complex

D pages 1020-1021

Which of the following differentiates an atrial rhythm from a sinus rhythm? A) Tachycardia B) Profound bradycardia C) Dissociated P waves D) Varying shapes in P waves

D pages 993

(2) when managing cardiac arrest, the appropriate dosing regimen for epinephrine is: A) 0.1 mg/kg of a 1:10,000 solution every 3 minutes B) 10 mL of a 1:1,000 solution every 3 to 5 minutes C) 1 mL of a 1:10,000 solution every 3 to 5 minutes D) 1 mg of a 1:10,000 solution every 3 to 5 minutes

D page 1016

Regardless of the patient's presenting cardiac arrest rhythm, the first IV or IO drug that should be given is: A) an inotrope B) calcium chloride C) an antidyshythmic D) vasopressor

D page 1016

A 70-year-old man called 911 because of generalized weakness. When you arrive at the scene, you find the patient seated in his recliner. He is conscious and alert and is breathing without difficulty. Your physical exam reveals tenderness to his right upper abdominal quadrant, edema to his ankles, and distended jugular veins. The patient tells you that he takes Vasotec for hypertension and Maxide for his swollen ankles. His vital sings are stable. The MOST appropriate treatment for this patient includes: A) high-flow oxygen, vascular access, 1 mg/kg of furosemide, and transport B) oxygen, cardiac monitoring, an IV line at a keep-open rate, and transport C) 12 lead ECG acquisiton, vascular access, 4 mg of morphine and transport D) an IV of D5W, 0.4 mg of sublingual nitroglycerin, ECG, and transport

B page 1051, 1054

In contrast to treatment for supraventricular tachycardia, treatment for multifocal atrial tachycardia in the prehospital setting: A) includes synchronized cardioversion B) involves atropine sulfate C) is generally not effective D) if often more effective

C page 1001

Which of the following is NOT characteristic of multifocal atrial tachycardia? A) QRS complexes less than 0.11 seconds in duration B) Noncisible P wave with a rapid ventricular rate C) Regular R-R intervals with a rate less than 150 beats/min D) Variable PR intervals and P waves of differing size

C page 1001

(1) a "run" of ventricular tachycardia occurs if at least ____ PVCs occur in a row. (quiz 2) A) four B) five C) three D) two

C page 1004

Premature ventricular complexes (PVCs) that originate from different sites in the ventricle: A) are also called fusion PVCs B) produce a palpable pulse C) will appear differently on the ECG D) are called unifocal PVCs

C page 1004

untreated ventricular tachycardia would MOST likely deteriorate to: A) asystole B) torsade de pointes C) ventricular fibrillation D) pulseless electrical activity

C page 1007

Common causes of cardiac arrest include all of the following, EXCPT: A) cardiac tamponade B) hypovolemia C C) hyperglycemia D) pulmonary embolism

C page 1014

You and an EMT are performing CPR on an elderly women in cardiac arrest as your paramedic partner prepares to intubate her. After the patient has been intubated and proper ET tube placement has been confirmed, you should: A) direct your partner to deliver one breath every 3 to 5 seconds as the EMT-B continues chest compression's B) instruct the EMT to pause after 30 compression's so your partner can deliver two ventilation's C) perform asynchronous CPR ventilating the patient at a rate of 10 D) administer 2.5 mg of epinephrine via the ET tube and hyperventilate the patient to ensure drug dispersal

C page 1014-1016

the proper compression-to-ventilation ratio for two-rescuer adult CPR when an oropharyngeal airway is in place is: A) 5:1 B) asynchronous C) 30:2 D) 15:2

C page 1016

A key to interpreting a Mobitz type II second-degree heart block is to remember that: A) most type II second degree AV blocks have more than two nonconducted P waves that occur in succession B) in this type of heart block, the PR interval gets progressively longer until a P wave is not conducted C) the PR interval of all the conducted P waves and their corresponding QRS complexes is constant D) unlike a Mobitz type I second-degree heart block, a type II heart block is always regular

C page 1020-1021

(4) a "runaway" pacemaker is characterized by: A) an absence of pacemaker spikes B) a narrowing of the QRS complexes C) a tachycardiac pacemaker rhythm D) profound slowing of the heart rate

C page 1023

When applying the precordial leads, lead V1 should be placed in the: A) fourth intercostal space at the left midclavicular border B) fifth intercostal space at the left midclavicular line C) fourth intercostal space at the right sternal border D) fourth intercostal space at the left sternal border

C page 1024

Aberrant conduction is ______ conduction A) very slow B) rapid C) abnormal D) irregular

C page 1027

The QRS in lead I is a negative deflection and the QRS in lead aVF is a positive deflection. this indicates: A) left axis deviation B) a normal axis C) right axis deviation D) extreme right axis deviation

C page 1027

A patient with right ventricular failure would most likely present with: A) pulmonary edema B) rhonchi in the lungs C) an enlarged liver D) collapsed jugular veins

C page 1051

A patient in cardiogenic shock without cardiac arrhythmias will benefit MOST from: A) a 250-mL bolus of a crystalloid solution B) supplemental oxygen C) rapid transport to an appropriate hospital D) a high-dose vasopressor infusion

C page 1057

A patient in cardiogenic shock without cardiac arrhythmias will benefit most from: A) a high-dose vasopressor infusion B) supplemental oxygen C) rapid transport to an appropriate hospital D) a 250 mL bolus of a crystalloid solution

C page 1057

A patient with cardiogenic shock and pulmonary edema should be positioned: A) in the Trendelenbrug position B) in a lateral recumbent position C) in a semi-Fowler position D) supine with legs elevated

C page 1057

Hypertension is present when the blood pressure: A) rises acutely during an emotionally stressful situation B) increases by 20 mm Hg above a person's normal blood pressure C) is consistently greater than 140/90 mm Hg while at rest D) is above 160 mm Hg systolic during strenuous exertion

C page 1058

Hypertensive disease is characterized by: (quiz 3) A) a systolic blood pressure greater than 140 mm Hg B) a diastolic blood pressure above 90 mm Hg C) persistent elevation of the diastolic pressure D) constant fluctuation in the systolic blood pressure

C page 1058

The process of aortic dissection begins when: A) blood accumulates between the layers of the aorta B) the aorta is weakened due to excessive pressure C) the intimal layer of the aortic wall is torn D) hypertension causes acute rupture of the aorta

C page 1062

A 70-year-old presents with pain in his legs while walking. Within a few minutes of sitting down, however, his symptoms resolve. What should you suspect? A) Right ventricular failure B) Abdominal aortic aneurysm C) Peripheral arterial disease D) Deep vein thrombosis

C page 1064

You are preforming CPR on an 80-year-old woman whose cardiac arrest was witnessed by her husband. Several intubation attempts have been unsuccessful, but ventilation's with a bag-mask device are producing adequate chest rise. IV access has been obtained and 1 mg og epinephrine has been administered. The cardiac monitor displays a narrow QRS complex rhythm at a rate of 70 beats/min. According to the patient's husband, she has had numerous episodes of diarrhea over the past 24 hours and has not had much of an appetite. The MOST appropriate next action should be to: A) administer 50% dextrose for presumed hypoglycemia B) administer 1 mg of atropine while CPR is ongoing C) assess the rhythm and pulse after 3 minutes of CPR D) continue CPR and administer crystalloid fluid boluses

D page 1017

The firing of an artificial ventricular pacemaker causes: A) a wide QRS complex followed by a vertical spike B) a change in the shape of the preceding P waves C) a small spike followed by a narrow QRS complex D) a vertical spike followed by a wide QRS complex

D page 1022

A patient's 12-lead ECG shows qR complexes in leads I and aVL and rS complexes in leads II, III, and aVF. This indicates: A)right bundle branch block B) left bundle branch block C) left posterior fascicular block D) left anterior fascicular block

D page 1027

On the 12-lead ECG, right atrial abnormality is characterized by: A) a P wave duration greater than 110 milliseconds in lead II B) a PR interval that is greater than 200 milliseconds C) negatively deflected P waves in leads II and V1 D) a P wave amplitude greater than 1.5 mm in lead V1

D page 1029

Lead I views the _____ wall of the heart, while lead aVF views the ____ wall of the heart. (quiz 3) A) anterior, inferior B) posterior, septal C) septal, anterior D) lateral, inferior

D page 1032

What 12-lead ECG finidng should make you suspect a posterior STEMI? A) ST depression in leads I and aVL B) ST elevation in leads III and aVF C) ST elevation in leads V3 and V4 D) ST depression in leads V1 and V2

D page 1032

Takotsubo cardiomyopathy is MOST often associated with: A) multiple coronary occlusions B) unexplained cardiac arrest C) a low blood pressure D) emotional stress

D page 1037

Atherosclerosis is process in which: A) the outer wall of a coronary artery becomes lined with masses of fatty tissue B) plaque ruptures from a distant location and lodges in one of the coronary arteries C) calcium precipitates into the arterial walls, greatly reducing the artery's elasticity D) plaque infiltrates the arterial wall, decreasing its elasticity and narrowing its lumen

D page 1039

Non-ST elevation myocardial infarction (NSTEMI): A) produces no signs of myocardial injury on the ECG B) produces no symptoms until permanent damage is done C) has a statistically worse outcome than ST elevation MI D) is not generally treated with cardiac catheterization

D page 1041

(6) the door-to-balloon time for a patient with an ST elevation myocardial infarction is ___ minutes or less. (quiz 2) A) 120 B) 30 C) 60 D) 90

D page 1044

In which of the following situations is oxygen indicated for a patient who complains of chest pain, pressure, or discomfort? A) The patient has a history of AMI B) oxygen saturation is less than 99% C) the 12 lead ECG indicates STEMI D) signs of heart failure are present

D page 1045

The MOST immediate forms of reperfusion therapy for an injured myocardium are: A) supplemental oxygen and an infusion of nitroflycerin B) angioplasty and coronary artery bypass grafting C) high-dose aspirin and high flow supplemental oxygen D) fibrionlytics and percutaneous coronary intervention

D page 1045

Which of the following statements regarding oxygen administration for a patient experiencing an acute myocardial infarction is correct? (quiz 3 ) A) evidence has shown that high (greater than 90%) concentrations of oxygen reduce mortality B) in order to prevent hypoxic injury, do not give any patient with an acute myocardial infarction more than 2L/min of oxygen C) any patient experiencing an acute myocardial infarction should receive high-flow oxygen D) treatment with oxygen should be individualized and titrated to maintain the SpO2 above 94%

D page 1045

You have just administered 0.4 mg of sublingual nitroglycerin to a 60-year-old woman with severe chest pain. The patient is receiving supplemental oxygen and has an IV line of normal saline in place. After 5 minutes, the patient states that the pain has not subsided. you should: A) administer 2 to 4 mg of morphine B) repeat the nitroglycerin C) give her 250 mL saline bolus D) reassess her blood pressure

D page 1045, 1047-1048

patients experiencing a right ventricular infarction: A) often require higher doses of morphine B) usually have anterior myocardial damage C) should not be given baby aspirin D) may present with hypotension

D page 1047-1048

Patients with a continuous-flow ventricular assist device: A) should be defibrillated with a higher-than-normal energy B) have a BP that is 20 to 30 mm Hg higher than is actually is C) cannot receive epinephrine if they are bradycardic D) may not have a palpable pulse, despite adequate perfusion

D page 1055

Cardiac tamponade can be differentiated from a tension pneumothorax by the presence of: A) narrowing pulse pressure B) alterations in the QRS amplitude C) jugular venous distention D) equal breath sounds

D page 1056

During the refractory period: A) the heart is in a state of partial repolarization B) the heart is partially charged, but cannot contact C) the heart muscle is depleted of energy and needs to recharge D) the cell is depolarized or in the process of repolarizing

D page 970

Hypomagnesemia would MOST likely result: A) decreased myocardial irritability B) a decrease in cardiac contractility C) decreased myocardial automaticity D) decreased cardiac conduction

D page 970

Which of the following electrolytes maintains the depolarization phase? A) sodium B) potassium C) magnesium D) calcium

D page 970

On the ECG graph paper, 6 seconds is represented by how many large boxes? A) 50 B) 40 C) 20 D) 30

D page 982


Conjuntos de estudio relacionados

FBLA Spreadsheet multiple choice

View Set

History 172 Quiz 2 Study Questions

View Set

fundamental principles of money vocabulary

View Set

Interest Rate and Loanable Funds

View Set

*13) Objective 3 Neuromuscular junction*

View Set

Fundamentals of nursing- walters kluwer chapter 1

View Set